Show that the following sequence converges. Please Critique my proof.

The name of the pictureThe name of the pictureThe name of the pictureClash Royale CLAN TAG#URR8PPP












9












$begingroup$


The problem is as follows:




Let $a_n$ be a sequence of nonnegative numbers such that
$$
a_n+1leq a_n+frac(-1)^nn.
$$

Show that $a_n$ converges.




My (wrong) proof:



Notice that
$$
|a_n+1-a_n|leq left|frac(-1)^nnright|leqfrac1n
$$

and since it is known that $frac1nrightarrow 0$ as $nrightarrow infty$. We see that we can arbitarily bound, $|a_n+1-a_n|$. Thus, $a_n$ converges.



My question:
This is a question from a comprehensive exam I found and am using to review.



Should I argue that we should select $N$ so that $n>N$ implies $left|frac1nright|<epsilon$ as well?



Notes: Currently working on the proof.










share|cite|improve this question











$endgroup$







  • 4




    $begingroup$
    Your proof is not correct. Your arguments would also work for $a_n = sum_i=1^n frac 1 i$, which does not converge.
    $endgroup$
    – Falrach
    Mar 1 at 21:13






  • 1




    $begingroup$
    Note that you not only need to bound $left| a_n+1 - a_n right|$ arbitrarily small, but also $left| a_m - a_n right|$ for all $m,n geq N$ (where $N$ can be chosen according to the bound).
    $endgroup$
    – Maximilian Janisch
    Mar 1 at 22:49







  • 1




    $begingroup$
    This is a duplicate, but I’m too lazy to find the original...
    $endgroup$
    – Shalop
    Mar 2 at 14:53










  • $begingroup$
    @Shalop if you do find the original, then please tell me.
    $endgroup$
    – Darel
    Mar 2 at 16:35















9












$begingroup$


The problem is as follows:




Let $a_n$ be a sequence of nonnegative numbers such that
$$
a_n+1leq a_n+frac(-1)^nn.
$$

Show that $a_n$ converges.




My (wrong) proof:



Notice that
$$
|a_n+1-a_n|leq left|frac(-1)^nnright|leqfrac1n
$$

and since it is known that $frac1nrightarrow 0$ as $nrightarrow infty$. We see that we can arbitarily bound, $|a_n+1-a_n|$. Thus, $a_n$ converges.



My question:
This is a question from a comprehensive exam I found and am using to review.



Should I argue that we should select $N$ so that $n>N$ implies $left|frac1nright|<epsilon$ as well?



Notes: Currently working on the proof.










share|cite|improve this question











$endgroup$







  • 4




    $begingroup$
    Your proof is not correct. Your arguments would also work for $a_n = sum_i=1^n frac 1 i$, which does not converge.
    $endgroup$
    – Falrach
    Mar 1 at 21:13






  • 1




    $begingroup$
    Note that you not only need to bound $left| a_n+1 - a_n right|$ arbitrarily small, but also $left| a_m - a_n right|$ for all $m,n geq N$ (where $N$ can be chosen according to the bound).
    $endgroup$
    – Maximilian Janisch
    Mar 1 at 22:49







  • 1




    $begingroup$
    This is a duplicate, but I’m too lazy to find the original...
    $endgroup$
    – Shalop
    Mar 2 at 14:53










  • $begingroup$
    @Shalop if you do find the original, then please tell me.
    $endgroup$
    – Darel
    Mar 2 at 16:35













9












9








9


1



$begingroup$


The problem is as follows:




Let $a_n$ be a sequence of nonnegative numbers such that
$$
a_n+1leq a_n+frac(-1)^nn.
$$

Show that $a_n$ converges.




My (wrong) proof:



Notice that
$$
|a_n+1-a_n|leq left|frac(-1)^nnright|leqfrac1n
$$

and since it is known that $frac1nrightarrow 0$ as $nrightarrow infty$. We see that we can arbitarily bound, $|a_n+1-a_n|$. Thus, $a_n$ converges.



My question:
This is a question from a comprehensive exam I found and am using to review.



Should I argue that we should select $N$ so that $n>N$ implies $left|frac1nright|<epsilon$ as well?



Notes: Currently working on the proof.










share|cite|improve this question











$endgroup$




The problem is as follows:




Let $a_n$ be a sequence of nonnegative numbers such that
$$
a_n+1leq a_n+frac(-1)^nn.
$$

Show that $a_n$ converges.




My (wrong) proof:



Notice that
$$
|a_n+1-a_n|leq left|frac(-1)^nnright|leqfrac1n
$$

and since it is known that $frac1nrightarrow 0$ as $nrightarrow infty$. We see that we can arbitarily bound, $|a_n+1-a_n|$. Thus, $a_n$ converges.



My question:
This is a question from a comprehensive exam I found and am using to review.



Should I argue that we should select $N$ so that $n>N$ implies $left|frac1nright|<epsilon$ as well?



Notes: Currently working on the proof.







real-analysis sequences-and-series convergence fake-proofs






share|cite|improve this question















share|cite|improve this question













share|cite|improve this question




share|cite|improve this question








edited Mar 2 at 1:37







Darel

















asked Mar 1 at 21:07









DarelDarel

1249




1249







  • 4




    $begingroup$
    Your proof is not correct. Your arguments would also work for $a_n = sum_i=1^n frac 1 i$, which does not converge.
    $endgroup$
    – Falrach
    Mar 1 at 21:13






  • 1




    $begingroup$
    Note that you not only need to bound $left| a_n+1 - a_n right|$ arbitrarily small, but also $left| a_m - a_n right|$ for all $m,n geq N$ (where $N$ can be chosen according to the bound).
    $endgroup$
    – Maximilian Janisch
    Mar 1 at 22:49







  • 1




    $begingroup$
    This is a duplicate, but I’m too lazy to find the original...
    $endgroup$
    – Shalop
    Mar 2 at 14:53










  • $begingroup$
    @Shalop if you do find the original, then please tell me.
    $endgroup$
    – Darel
    Mar 2 at 16:35












  • 4




    $begingroup$
    Your proof is not correct. Your arguments would also work for $a_n = sum_i=1^n frac 1 i$, which does not converge.
    $endgroup$
    – Falrach
    Mar 1 at 21:13






  • 1




    $begingroup$
    Note that you not only need to bound $left| a_n+1 - a_n right|$ arbitrarily small, but also $left| a_m - a_n right|$ for all $m,n geq N$ (where $N$ can be chosen according to the bound).
    $endgroup$
    – Maximilian Janisch
    Mar 1 at 22:49







  • 1




    $begingroup$
    This is a duplicate, but I’m too lazy to find the original...
    $endgroup$
    – Shalop
    Mar 2 at 14:53










  • $begingroup$
    @Shalop if you do find the original, then please tell me.
    $endgroup$
    – Darel
    Mar 2 at 16:35







4




4




$begingroup$
Your proof is not correct. Your arguments would also work for $a_n = sum_i=1^n frac 1 i$, which does not converge.
$endgroup$
– Falrach
Mar 1 at 21:13




$begingroup$
Your proof is not correct. Your arguments would also work for $a_n = sum_i=1^n frac 1 i$, which does not converge.
$endgroup$
– Falrach
Mar 1 at 21:13




1




1




$begingroup$
Note that you not only need to bound $left| a_n+1 - a_n right|$ arbitrarily small, but also $left| a_m - a_n right|$ for all $m,n geq N$ (where $N$ can be chosen according to the bound).
$endgroup$
– Maximilian Janisch
Mar 1 at 22:49





$begingroup$
Note that you not only need to bound $left| a_n+1 - a_n right|$ arbitrarily small, but also $left| a_m - a_n right|$ for all $m,n geq N$ (where $N$ can be chosen according to the bound).
$endgroup$
– Maximilian Janisch
Mar 1 at 22:49





1




1




$begingroup$
This is a duplicate, but I’m too lazy to find the original...
$endgroup$
– Shalop
Mar 2 at 14:53




$begingroup$
This is a duplicate, but I’m too lazy to find the original...
$endgroup$
– Shalop
Mar 2 at 14:53












$begingroup$
@Shalop if you do find the original, then please tell me.
$endgroup$
– Darel
Mar 2 at 16:35




$begingroup$
@Shalop if you do find the original, then please tell me.
$endgroup$
– Darel
Mar 2 at 16:35










2 Answers
2






active

oldest

votes


















5












$begingroup$

Consider $b_n = a_n + sum_k=1^n-1 frac(-1)^k-1k$. Then



$$ b_n+1
= a_n+1 + sum_k=1^n frac(-1)^k-1k
leq a_n + frac(-1)^nn + sum_k=1^n frac(-1)^k-1k
= b_n, $$



which shows that $(b_n)$ is non-increasing. Moreover, since $sum_k=1^infty frac(-1)^k-1k$ converges by alternating series test and $(a_n)$ is non-negative, it follows that $(b_n)$ is bounded from below. Therefore $(b_n)$ converges, and so, $(a_n)$ converges as well.






share|cite|improve this answer









$endgroup$








  • 3




    $begingroup$
    Thank you, that's neat! One might add that this argument always works for lower-bounded $(a_n)$ with $a_n+1le a_n+c_n$ for some summable $(c_n)$ by setting $b_n=a_n-sum_k=1^n-1c_k$.
    $endgroup$
    – Mars Plastic
    Mar 1 at 23:04











  • $begingroup$
    @MarsPlastic The argument works even if $(a_n)$ is not bounded below, in that case $a_n to -infty$ follows.
    $endgroup$
    – Martin R
    Mar 2 at 7:28










  • $begingroup$
    @MarsPlastic: Btw, thank you for pointing out the flaw in my answer. I was able to fix that, but this answer is so elegant and much simpler, that I deleted mine again.
    $endgroup$
    – Martin R
    Mar 2 at 8:04






  • 1




    $begingroup$
    This is really an elegant way. Unfortunately I just got it done by brute force.
    $endgroup$
    – Falrach
    Mar 2 at 8:36


















1












$begingroup$

Define $b_k := a_2k+1$. Then
$$b_k leq a_2k + (-1)^2kfrac12k leq b_k-1 + (frac12k - frac12k-1) leq b_k-1$$
Since $b_k$ is non-negative and non-increasing: $b_k to b$.
Suppose $a_n nrightarrow b$. Then there exists an $varepsilon > 0 $ s.t. for infinitely many $n$ holds $|a_2n - b| > varepsilon$.
Assume that $|a_2m+1-a_m| > fracvarepsilon2$ for infinitely many $m$. Then, since $a_2m+1- a_m leq frac12m$ we have that
beginalign
a_2m+1 - a_m < - fracvarepsilon2
endalign

for infinitely many $m$. Let $M := m geq 1 : a_2m+1 - a_m < - fracvarepsilon2 text is fulfilled for m $
beginalign*
d_m := 1_M (m)
endalign*

This implies
beginalign*
0 leq a_2m+1 = a_1 + sum_k=1^2m (a_k+1 - a_k ) = a_1 + sum_k=1^m (a_2k+1 - a_2k) + sum_k=1^m (a_2k - a_2k-1) \
leq a_1 + sum_k=1^m (-1)^2k frac12k- fracvarepsilon2 d_k + sum_k=1^m (-1)^2k-1frac12k-1 to a_1 - sum_k=1^infty fracvarepsilon2 d_k + sum_i=1^infty (-1)^i frac1i = - infty
endalign*

since $|M| = infty$ and the last series converges. This is a contradiction.
Therefore we have that there exists $Kgeq 1$ s.t. for all $kgeq K$ it holds: $|a_2k+1 - a_k| leq fracvarepsilon2$. We can conclude that
beginalign*
|a2n+1 - b| geq |a_2n - b| - |a_2n+1 - a_n| geq varepsilon - fracvarepsilon2 = fracvarepsilon2
endalign*

for infinitely $n geq K$. Contradiction. Thus $a_n to b$.






share|cite|improve this answer









$endgroup$













    Your Answer





    StackExchange.ifUsing("editor", function ()
    return StackExchange.using("mathjaxEditing", function ()
    StackExchange.MarkdownEditor.creationCallbacks.add(function (editor, postfix)
    StackExchange.mathjaxEditing.prepareWmdForMathJax(editor, postfix, [["$", "$"], ["\\(","\\)"]]);
    );
    );
    , "mathjax-editing");

    StackExchange.ready(function()
    var channelOptions =
    tags: "".split(" "),
    id: "69"
    ;
    initTagRenderer("".split(" "), "".split(" "), channelOptions);

    StackExchange.using("externalEditor", function()
    // Have to fire editor after snippets, if snippets enabled
    if (StackExchange.settings.snippets.snippetsEnabled)
    StackExchange.using("snippets", function()
    createEditor();
    );

    else
    createEditor();

    );

    function createEditor()
    StackExchange.prepareEditor(
    heartbeatType: 'answer',
    autoActivateHeartbeat: false,
    convertImagesToLinks: true,
    noModals: true,
    showLowRepImageUploadWarning: true,
    reputationToPostImages: 10,
    bindNavPrevention: true,
    postfix: "",
    imageUploader:
    brandingHtml: "Powered by u003ca class="icon-imgur-white" href="https://imgur.com/"u003eu003c/au003e",
    contentPolicyHtml: "User contributions licensed under u003ca href="https://creativecommons.org/licenses/by-sa/3.0/"u003ecc by-sa 3.0 with attribution requiredu003c/au003e u003ca href="https://stackoverflow.com/legal/content-policy"u003e(content policy)u003c/au003e",
    allowUrls: true
    ,
    noCode: true, onDemand: true,
    discardSelector: ".discard-answer"
    ,immediatelyShowMarkdownHelp:true
    );



    );













    draft saved

    draft discarded


















    StackExchange.ready(
    function ()
    StackExchange.openid.initPostLogin('.new-post-login', 'https%3a%2f%2fmath.stackexchange.com%2fquestions%2f3131816%2fshow-that-the-following-sequence-converges-please-critique-my-proof%23new-answer', 'question_page');

    );

    Post as a guest















    Required, but never shown

























    2 Answers
    2






    active

    oldest

    votes








    2 Answers
    2






    active

    oldest

    votes









    active

    oldest

    votes






    active

    oldest

    votes









    5












    $begingroup$

    Consider $b_n = a_n + sum_k=1^n-1 frac(-1)^k-1k$. Then



    $$ b_n+1
    = a_n+1 + sum_k=1^n frac(-1)^k-1k
    leq a_n + frac(-1)^nn + sum_k=1^n frac(-1)^k-1k
    = b_n, $$



    which shows that $(b_n)$ is non-increasing. Moreover, since $sum_k=1^infty frac(-1)^k-1k$ converges by alternating series test and $(a_n)$ is non-negative, it follows that $(b_n)$ is bounded from below. Therefore $(b_n)$ converges, and so, $(a_n)$ converges as well.






    share|cite|improve this answer









    $endgroup$








    • 3




      $begingroup$
      Thank you, that's neat! One might add that this argument always works for lower-bounded $(a_n)$ with $a_n+1le a_n+c_n$ for some summable $(c_n)$ by setting $b_n=a_n-sum_k=1^n-1c_k$.
      $endgroup$
      – Mars Plastic
      Mar 1 at 23:04











    • $begingroup$
      @MarsPlastic The argument works even if $(a_n)$ is not bounded below, in that case $a_n to -infty$ follows.
      $endgroup$
      – Martin R
      Mar 2 at 7:28










    • $begingroup$
      @MarsPlastic: Btw, thank you for pointing out the flaw in my answer. I was able to fix that, but this answer is so elegant and much simpler, that I deleted mine again.
      $endgroup$
      – Martin R
      Mar 2 at 8:04






    • 1




      $begingroup$
      This is really an elegant way. Unfortunately I just got it done by brute force.
      $endgroup$
      – Falrach
      Mar 2 at 8:36















    5












    $begingroup$

    Consider $b_n = a_n + sum_k=1^n-1 frac(-1)^k-1k$. Then



    $$ b_n+1
    = a_n+1 + sum_k=1^n frac(-1)^k-1k
    leq a_n + frac(-1)^nn + sum_k=1^n frac(-1)^k-1k
    = b_n, $$



    which shows that $(b_n)$ is non-increasing. Moreover, since $sum_k=1^infty frac(-1)^k-1k$ converges by alternating series test and $(a_n)$ is non-negative, it follows that $(b_n)$ is bounded from below. Therefore $(b_n)$ converges, and so, $(a_n)$ converges as well.






    share|cite|improve this answer









    $endgroup$








    • 3




      $begingroup$
      Thank you, that's neat! One might add that this argument always works for lower-bounded $(a_n)$ with $a_n+1le a_n+c_n$ for some summable $(c_n)$ by setting $b_n=a_n-sum_k=1^n-1c_k$.
      $endgroup$
      – Mars Plastic
      Mar 1 at 23:04











    • $begingroup$
      @MarsPlastic The argument works even if $(a_n)$ is not bounded below, in that case $a_n to -infty$ follows.
      $endgroup$
      – Martin R
      Mar 2 at 7:28










    • $begingroup$
      @MarsPlastic: Btw, thank you for pointing out the flaw in my answer. I was able to fix that, but this answer is so elegant and much simpler, that I deleted mine again.
      $endgroup$
      – Martin R
      Mar 2 at 8:04






    • 1




      $begingroup$
      This is really an elegant way. Unfortunately I just got it done by brute force.
      $endgroup$
      – Falrach
      Mar 2 at 8:36













    5












    5








    5





    $begingroup$

    Consider $b_n = a_n + sum_k=1^n-1 frac(-1)^k-1k$. Then



    $$ b_n+1
    = a_n+1 + sum_k=1^n frac(-1)^k-1k
    leq a_n + frac(-1)^nn + sum_k=1^n frac(-1)^k-1k
    = b_n, $$



    which shows that $(b_n)$ is non-increasing. Moreover, since $sum_k=1^infty frac(-1)^k-1k$ converges by alternating series test and $(a_n)$ is non-negative, it follows that $(b_n)$ is bounded from below. Therefore $(b_n)$ converges, and so, $(a_n)$ converges as well.






    share|cite|improve this answer









    $endgroup$



    Consider $b_n = a_n + sum_k=1^n-1 frac(-1)^k-1k$. Then



    $$ b_n+1
    = a_n+1 + sum_k=1^n frac(-1)^k-1k
    leq a_n + frac(-1)^nn + sum_k=1^n frac(-1)^k-1k
    = b_n, $$



    which shows that $(b_n)$ is non-increasing. Moreover, since $sum_k=1^infty frac(-1)^k-1k$ converges by alternating series test and $(a_n)$ is non-negative, it follows that $(b_n)$ is bounded from below. Therefore $(b_n)$ converges, and so, $(a_n)$ converges as well.







    share|cite|improve this answer












    share|cite|improve this answer



    share|cite|improve this answer










    answered Mar 1 at 22:47









    Sangchul LeeSangchul Lee

    96.2k12171282




    96.2k12171282







    • 3




      $begingroup$
      Thank you, that's neat! One might add that this argument always works for lower-bounded $(a_n)$ with $a_n+1le a_n+c_n$ for some summable $(c_n)$ by setting $b_n=a_n-sum_k=1^n-1c_k$.
      $endgroup$
      – Mars Plastic
      Mar 1 at 23:04











    • $begingroup$
      @MarsPlastic The argument works even if $(a_n)$ is not bounded below, in that case $a_n to -infty$ follows.
      $endgroup$
      – Martin R
      Mar 2 at 7:28










    • $begingroup$
      @MarsPlastic: Btw, thank you for pointing out the flaw in my answer. I was able to fix that, but this answer is so elegant and much simpler, that I deleted mine again.
      $endgroup$
      – Martin R
      Mar 2 at 8:04






    • 1




      $begingroup$
      This is really an elegant way. Unfortunately I just got it done by brute force.
      $endgroup$
      – Falrach
      Mar 2 at 8:36












    • 3




      $begingroup$
      Thank you, that's neat! One might add that this argument always works for lower-bounded $(a_n)$ with $a_n+1le a_n+c_n$ for some summable $(c_n)$ by setting $b_n=a_n-sum_k=1^n-1c_k$.
      $endgroup$
      – Mars Plastic
      Mar 1 at 23:04











    • $begingroup$
      @MarsPlastic The argument works even if $(a_n)$ is not bounded below, in that case $a_n to -infty$ follows.
      $endgroup$
      – Martin R
      Mar 2 at 7:28










    • $begingroup$
      @MarsPlastic: Btw, thank you for pointing out the flaw in my answer. I was able to fix that, but this answer is so elegant and much simpler, that I deleted mine again.
      $endgroup$
      – Martin R
      Mar 2 at 8:04






    • 1




      $begingroup$
      This is really an elegant way. Unfortunately I just got it done by brute force.
      $endgroup$
      – Falrach
      Mar 2 at 8:36







    3




    3




    $begingroup$
    Thank you, that's neat! One might add that this argument always works for lower-bounded $(a_n)$ with $a_n+1le a_n+c_n$ for some summable $(c_n)$ by setting $b_n=a_n-sum_k=1^n-1c_k$.
    $endgroup$
    – Mars Plastic
    Mar 1 at 23:04





    $begingroup$
    Thank you, that's neat! One might add that this argument always works for lower-bounded $(a_n)$ with $a_n+1le a_n+c_n$ for some summable $(c_n)$ by setting $b_n=a_n-sum_k=1^n-1c_k$.
    $endgroup$
    – Mars Plastic
    Mar 1 at 23:04













    $begingroup$
    @MarsPlastic The argument works even if $(a_n)$ is not bounded below, in that case $a_n to -infty$ follows.
    $endgroup$
    – Martin R
    Mar 2 at 7:28




    $begingroup$
    @MarsPlastic The argument works even if $(a_n)$ is not bounded below, in that case $a_n to -infty$ follows.
    $endgroup$
    – Martin R
    Mar 2 at 7:28












    $begingroup$
    @MarsPlastic: Btw, thank you for pointing out the flaw in my answer. I was able to fix that, but this answer is so elegant and much simpler, that I deleted mine again.
    $endgroup$
    – Martin R
    Mar 2 at 8:04




    $begingroup$
    @MarsPlastic: Btw, thank you for pointing out the flaw in my answer. I was able to fix that, but this answer is so elegant and much simpler, that I deleted mine again.
    $endgroup$
    – Martin R
    Mar 2 at 8:04




    1




    1




    $begingroup$
    This is really an elegant way. Unfortunately I just got it done by brute force.
    $endgroup$
    – Falrach
    Mar 2 at 8:36




    $begingroup$
    This is really an elegant way. Unfortunately I just got it done by brute force.
    $endgroup$
    – Falrach
    Mar 2 at 8:36











    1












    $begingroup$

    Define $b_k := a_2k+1$. Then
    $$b_k leq a_2k + (-1)^2kfrac12k leq b_k-1 + (frac12k - frac12k-1) leq b_k-1$$
    Since $b_k$ is non-negative and non-increasing: $b_k to b$.
    Suppose $a_n nrightarrow b$. Then there exists an $varepsilon > 0 $ s.t. for infinitely many $n$ holds $|a_2n - b| > varepsilon$.
    Assume that $|a_2m+1-a_m| > fracvarepsilon2$ for infinitely many $m$. Then, since $a_2m+1- a_m leq frac12m$ we have that
    beginalign
    a_2m+1 - a_m < - fracvarepsilon2
    endalign

    for infinitely many $m$. Let $M := m geq 1 : a_2m+1 - a_m < - fracvarepsilon2 text is fulfilled for m $
    beginalign*
    d_m := 1_M (m)
    endalign*

    This implies
    beginalign*
    0 leq a_2m+1 = a_1 + sum_k=1^2m (a_k+1 - a_k ) = a_1 + sum_k=1^m (a_2k+1 - a_2k) + sum_k=1^m (a_2k - a_2k-1) \
    leq a_1 + sum_k=1^m (-1)^2k frac12k- fracvarepsilon2 d_k + sum_k=1^m (-1)^2k-1frac12k-1 to a_1 - sum_k=1^infty fracvarepsilon2 d_k + sum_i=1^infty (-1)^i frac1i = - infty
    endalign*

    since $|M| = infty$ and the last series converges. This is a contradiction.
    Therefore we have that there exists $Kgeq 1$ s.t. for all $kgeq K$ it holds: $|a_2k+1 - a_k| leq fracvarepsilon2$. We can conclude that
    beginalign*
    |a2n+1 - b| geq |a_2n - b| - |a_2n+1 - a_n| geq varepsilon - fracvarepsilon2 = fracvarepsilon2
    endalign*

    for infinitely $n geq K$. Contradiction. Thus $a_n to b$.






    share|cite|improve this answer









    $endgroup$

















      1












      $begingroup$

      Define $b_k := a_2k+1$. Then
      $$b_k leq a_2k + (-1)^2kfrac12k leq b_k-1 + (frac12k - frac12k-1) leq b_k-1$$
      Since $b_k$ is non-negative and non-increasing: $b_k to b$.
      Suppose $a_n nrightarrow b$. Then there exists an $varepsilon > 0 $ s.t. for infinitely many $n$ holds $|a_2n - b| > varepsilon$.
      Assume that $|a_2m+1-a_m| > fracvarepsilon2$ for infinitely many $m$. Then, since $a_2m+1- a_m leq frac12m$ we have that
      beginalign
      a_2m+1 - a_m < - fracvarepsilon2
      endalign

      for infinitely many $m$. Let $M := m geq 1 : a_2m+1 - a_m < - fracvarepsilon2 text is fulfilled for m $
      beginalign*
      d_m := 1_M (m)
      endalign*

      This implies
      beginalign*
      0 leq a_2m+1 = a_1 + sum_k=1^2m (a_k+1 - a_k ) = a_1 + sum_k=1^m (a_2k+1 - a_2k) + sum_k=1^m (a_2k - a_2k-1) \
      leq a_1 + sum_k=1^m (-1)^2k frac12k- fracvarepsilon2 d_k + sum_k=1^m (-1)^2k-1frac12k-1 to a_1 - sum_k=1^infty fracvarepsilon2 d_k + sum_i=1^infty (-1)^i frac1i = - infty
      endalign*

      since $|M| = infty$ and the last series converges. This is a contradiction.
      Therefore we have that there exists $Kgeq 1$ s.t. for all $kgeq K$ it holds: $|a_2k+1 - a_k| leq fracvarepsilon2$. We can conclude that
      beginalign*
      |a2n+1 - b| geq |a_2n - b| - |a_2n+1 - a_n| geq varepsilon - fracvarepsilon2 = fracvarepsilon2
      endalign*

      for infinitely $n geq K$. Contradiction. Thus $a_n to b$.






      share|cite|improve this answer









      $endgroup$















        1












        1








        1





        $begingroup$

        Define $b_k := a_2k+1$. Then
        $$b_k leq a_2k + (-1)^2kfrac12k leq b_k-1 + (frac12k - frac12k-1) leq b_k-1$$
        Since $b_k$ is non-negative and non-increasing: $b_k to b$.
        Suppose $a_n nrightarrow b$. Then there exists an $varepsilon > 0 $ s.t. for infinitely many $n$ holds $|a_2n - b| > varepsilon$.
        Assume that $|a_2m+1-a_m| > fracvarepsilon2$ for infinitely many $m$. Then, since $a_2m+1- a_m leq frac12m$ we have that
        beginalign
        a_2m+1 - a_m < - fracvarepsilon2
        endalign

        for infinitely many $m$. Let $M := m geq 1 : a_2m+1 - a_m < - fracvarepsilon2 text is fulfilled for m $
        beginalign*
        d_m := 1_M (m)
        endalign*

        This implies
        beginalign*
        0 leq a_2m+1 = a_1 + sum_k=1^2m (a_k+1 - a_k ) = a_1 + sum_k=1^m (a_2k+1 - a_2k) + sum_k=1^m (a_2k - a_2k-1) \
        leq a_1 + sum_k=1^m (-1)^2k frac12k- fracvarepsilon2 d_k + sum_k=1^m (-1)^2k-1frac12k-1 to a_1 - sum_k=1^infty fracvarepsilon2 d_k + sum_i=1^infty (-1)^i frac1i = - infty
        endalign*

        since $|M| = infty$ and the last series converges. This is a contradiction.
        Therefore we have that there exists $Kgeq 1$ s.t. for all $kgeq K$ it holds: $|a_2k+1 - a_k| leq fracvarepsilon2$. We can conclude that
        beginalign*
        |a2n+1 - b| geq |a_2n - b| - |a_2n+1 - a_n| geq varepsilon - fracvarepsilon2 = fracvarepsilon2
        endalign*

        for infinitely $n geq K$. Contradiction. Thus $a_n to b$.






        share|cite|improve this answer









        $endgroup$



        Define $b_k := a_2k+1$. Then
        $$b_k leq a_2k + (-1)^2kfrac12k leq b_k-1 + (frac12k - frac12k-1) leq b_k-1$$
        Since $b_k$ is non-negative and non-increasing: $b_k to b$.
        Suppose $a_n nrightarrow b$. Then there exists an $varepsilon > 0 $ s.t. for infinitely many $n$ holds $|a_2n - b| > varepsilon$.
        Assume that $|a_2m+1-a_m| > fracvarepsilon2$ for infinitely many $m$. Then, since $a_2m+1- a_m leq frac12m$ we have that
        beginalign
        a_2m+1 - a_m < - fracvarepsilon2
        endalign

        for infinitely many $m$. Let $M := m geq 1 : a_2m+1 - a_m < - fracvarepsilon2 text is fulfilled for m $
        beginalign*
        d_m := 1_M (m)
        endalign*

        This implies
        beginalign*
        0 leq a_2m+1 = a_1 + sum_k=1^2m (a_k+1 - a_k ) = a_1 + sum_k=1^m (a_2k+1 - a_2k) + sum_k=1^m (a_2k - a_2k-1) \
        leq a_1 + sum_k=1^m (-1)^2k frac12k- fracvarepsilon2 d_k + sum_k=1^m (-1)^2k-1frac12k-1 to a_1 - sum_k=1^infty fracvarepsilon2 d_k + sum_i=1^infty (-1)^i frac1i = - infty
        endalign*

        since $|M| = infty$ and the last series converges. This is a contradiction.
        Therefore we have that there exists $Kgeq 1$ s.t. for all $kgeq K$ it holds: $|a_2k+1 - a_k| leq fracvarepsilon2$. We can conclude that
        beginalign*
        |a2n+1 - b| geq |a_2n - b| - |a_2n+1 - a_n| geq varepsilon - fracvarepsilon2 = fracvarepsilon2
        endalign*

        for infinitely $n geq K$. Contradiction. Thus $a_n to b$.







        share|cite|improve this answer












        share|cite|improve this answer



        share|cite|improve this answer










        answered Mar 2 at 1:11









        FalrachFalrach

        1,745225




        1,745225



























            draft saved

            draft discarded
















































            Thanks for contributing an answer to Mathematics Stack Exchange!


            • Please be sure to answer the question. Provide details and share your research!

            But avoid


            • Asking for help, clarification, or responding to other answers.

            • Making statements based on opinion; back them up with references or personal experience.

            Use MathJax to format equations. MathJax reference.


            To learn more, see our tips on writing great answers.




            draft saved


            draft discarded














            StackExchange.ready(
            function ()
            StackExchange.openid.initPostLogin('.new-post-login', 'https%3a%2f%2fmath.stackexchange.com%2fquestions%2f3131816%2fshow-that-the-following-sequence-converges-please-critique-my-proof%23new-answer', 'question_page');

            );

            Post as a guest















            Required, but never shown





















































            Required, but never shown














            Required, but never shown












            Required, but never shown







            Required, but never shown

































            Required, but never shown














            Required, but never shown












            Required, but never shown







            Required, but never shown






            Popular posts from this blog

            How to check contact read email or not when send email to Individual?

            Bahrain

            Postfix configuration issue with fips on centos 7; mailgun relay